Difference between revisions of "2013 AIME II Problems/Problem 4"

(Problem 4)
Line 21: Line 21:
  
 
We can then use the Pythagorean Theorem on line segment <math>MP</math>: <math>MP^2 = \Delta x^2 + \Delta y^2</math> yields <math>\Delta y = -\frac{1}{2\sqrt{3}}</math> and <math>\Delta x = 1</math>, after substituting <math>\Delta x</math>. The coordinates of P are thus <math>\left(\frac{5}{2},\ \frac{5\sqrt{3}}{6}\right)</math>. Multiplying these together gives us <math>\frac{25\sqrt{3}}{12}</math>, giving us <math>\boxed{040}</math> as our answer.
 
We can then use the Pythagorean Theorem on line segment <math>MP</math>: <math>MP^2 = \Delta x^2 + \Delta y^2</math> yields <math>\Delta y = -\frac{1}{2\sqrt{3}}</math> and <math>\Delta x = 1</math>, after substituting <math>\Delta x</math>. The coordinates of P are thus <math>\left(\frac{5}{2},\ \frac{5\sqrt{3}}{6}\right)</math>. Multiplying these together gives us <math>\frac{25\sqrt{3}}{12}</math>, giving us <math>\boxed{040}</math> as our answer.
 +
 +
==Solution 4==
 +
Since <math>AC</math> will be segment <math>AB</math> rotated clockwise <math>60^{\circ}</math>, we can use a rotation matrix to find <math>C</math>. We first translate the triangle <math>1</math> unit to the left, so <math>A'</math> lies on the origin, and <math>B' = (1, 2\sqrt{3})</math>. Rotating clockwise <math>60^{\circ}</math> is the same as rotating <math>300^{\circ}</math> counter-clockwise, so our rotation matrix is <math>\begin{bmatrix} \cos{300^{\circ}} & -\sin{300^{\circ}}\\ \sin{300^{\circ}} & \cos{300^{\circ}}\\ \end{bmatrix} = \begin{bmatrix} \frac{1}{2} & \frac{\sqrt{3}}{2}\\ -\frac{\sqrt{3}}{2} & \frac{1}{2}\\ \end{bmatrix}</math>. Then <math>C' = \begin{bmatrix} \frac{1}{2} & \frac{\sqrt{3}}{2}\\ -\frac{\sqrt{3}}{2} & \frac{1}{2}\\ \end{bmatrix} \cdot \begin{bmatrix} 1\\ 2\sqrt{3}\\ \end{bmatrix} = \begin{bmatrix} \frac{7}{2}\\ \frac{\sqrt{3}}{2}\\ \end{bmatrix}</math>. Thus, <math>C = (\frac{9}{2}, \frac{\sqrt{3}}{2})</math>. Since the triangle is equilateral, the center of the triangle is the average of the coordinates of the vertices. Then <math>P = (\frac{1 + 2 + \frac{9}{2}}{3}, \frac{2\sqrt{3} + \frac{\sqrt{3}}{2}}{3}) = (\frac{5}{2}, \frac{5\sqrt{3}}{6})</math>. Our answer is <math>\frac{5}{2} \cdot \frac{5\sqrt{3}}{6} = \frac{25\sqrt{3}}{12}</math>. <math>25 + 3 + 12 = \boxed{40}</math>
  
 
==See Also==
 
==See Also==
 
{{AIME box|year=2013|n=II|num-b=3|num-a=5}}
 
{{AIME box|year=2013|n=II|num-b=3|num-a=5}}
 
{{MAA Notice}}
 
{{MAA Notice}}

Revision as of 15:44, 14 March 2016

Problem

In the Cartesian plane let $A = (1,0)$ and $B = \left( 2, 2\sqrt{3} \right)$. Equilateral triangle $ABC$ is constructed so that $C$ lies in the first quadrant. Let $P=(x,y)$ be the center of $\triangle ABC$. Then $x \cdot y$ can be written as $\tfrac{p\sqrt{q}}{r}$, where $p$ and $r$ are relatively prime positive integers and $q$ is an integer that is not divisible by the square of any prime. Find $p+q+r$.

Solution 1

The distance from point $A$ to point $B$ is $\sqrt{13}$. The vector that starts at point A and ends at point B is given by $B - A = (1, 2\sqrt{3})$. Since the center of an equilateral triangle, $P$, is also the intersection of the perpendicular bisectors of the sides of the triangle, we need first find the equation for the perpendicular bisector to $\overline{AB}$. The line perpendicular to $\overline{AB}$ through the midpoint, $M =  (\dfrac{3}{2},\sqrt{3})$, $\overline{AB}$ can be parameterized by $(\dfrac{2\sqrt{3}}{\sqrt{13}}, \dfrac{-1}{\sqrt{13}}) t + (\dfrac{3}{2},\sqrt{3})$. At this point, it is useful to note that $\Delta BMP$ is a 30-60-90 triangle with $\overline{MB}$ measuring $\dfrac{\sqrt{13}}{2}$. This yields the length of $\overline{MP}$ to be $\dfrac{\sqrt{13}}{2\sqrt{3}}$. Therefore, $P =( \dfrac{2\sqrt{3}}{\sqrt{13}},\dfrac{-1}{\sqrt{13}})(\dfrac{\sqrt{13}}{2\sqrt{3}}) + (\dfrac{3}{2},\sqrt{3}) = (\dfrac{5}{2}, \dfrac{5}{2\sqrt{3}})$. Therefore $xy = \dfrac{25\sqrt{3}}{12}$ yielding an answer of $p + q + r  = 25 + 3 + 12 = \boxed{040}$.


Solution 2

Rather than considering the Cartesian plane, we use complex numbers. Thus A is 1 and B is $2 + 2\sqrt{3}i$.

Recall that a rotation of $\theta$ radians counterclockwise is equivalent to multiplying a complex number by $e^{i\theta}$, but here we require a clockwise rotation, so we multiply by $e^{-\frac{i\pi}{3}}$ to obtain C. Upon averaging the coordinates of A, B, and C, we obtain the coordinates of P, viz. $\left(\frac{5}{2}, \frac{5\sqrt{3}}{6}\right)$.

Therefore $xy$ is $\frac{25\sqrt{3}}{12}$ and the answer is $25 + 12 + 3 = \boxed{040}$.

Solution 3

We can also consider the slopes of the lines. Midpoint $M$ of $AB$ has coordinates $\left(\frac{3}{2},\ \sqrt{3}\right)$. Because line $AB$ has slope $2\sqrt{3}$, the slope of line $MP$ is $-\frac{1}{2\sqrt{3}}$.

Since $\Delta ABC$ is equilateral, and since point $P$ is the centroid, we can quickly calculate that $MP = \frac{\sqrt{39}}{6}$. Then, define $\Delta x$ and $\Delta y$ to be the differences between points $M$ and $P$. Because of the slope, it is clear that $\Delta x = 2\sqrt{3} \Delta y$.

We can then use the Pythagorean Theorem on line segment $MP$: $MP^2 = \Delta x^2 + \Delta y^2$ yields $\Delta y = -\frac{1}{2\sqrt{3}}$ and $\Delta x = 1$, after substituting $\Delta x$. The coordinates of P are thus $\left(\frac{5}{2},\ \frac{5\sqrt{3}}{6}\right)$. Multiplying these together gives us $\frac{25\sqrt{3}}{12}$, giving us $\boxed{040}$ as our answer.

Solution 4

Since $AC$ will be segment $AB$ rotated clockwise $60^{\circ}$, we can use a rotation matrix to find $C$. We first translate the triangle $1$ unit to the left, so $A'$ lies on the origin, and $B' = (1, 2\sqrt{3})$. Rotating clockwise $60^{\circ}$ is the same as rotating $300^{\circ}$ counter-clockwise, so our rotation matrix is $\begin{bmatrix} \cos{300^{\circ}} & -\sin{300^{\circ}}\\ \sin{300^{\circ}} & \cos{300^{\circ}}\\ \end{bmatrix} = \begin{bmatrix} \frac{1}{2} & \frac{\sqrt{3}}{2}\\ -\frac{\sqrt{3}}{2} & \frac{1}{2}\\ \end{bmatrix}$. Then $C' = \begin{bmatrix} \frac{1}{2} & \frac{\sqrt{3}}{2}\\ -\frac{\sqrt{3}}{2} & \frac{1}{2}\\ \end{bmatrix} \cdot \begin{bmatrix} 1\\ 2\sqrt{3}\\ \end{bmatrix} = \begin{bmatrix} \frac{7}{2}\\ \frac{\sqrt{3}}{2}\\ \end{bmatrix}$. Thus, $C = (\frac{9}{2}, \frac{\sqrt{3}}{2})$. Since the triangle is equilateral, the center of the triangle is the average of the coordinates of the vertices. Then $P = (\frac{1 + 2 + \frac{9}{2}}{3}, \frac{2\sqrt{3} + \frac{\sqrt{3}}{2}}{3}) = (\frac{5}{2}, \frac{5\sqrt{3}}{6})$. Our answer is $\frac{5}{2} \cdot \frac{5\sqrt{3}}{6} = \frac{25\sqrt{3}}{12}$. $25 + 3 + 12 = \boxed{40}$

See Also

2013 AIME II (ProblemsAnswer KeyResources)
Preceded by
Problem 3
Followed by
Problem 5
1 2 3 4 5 6 7 8 9 10 11 12 13 14 15
All AIME Problems and Solutions

The problems on this page are copyrighted by the Mathematical Association of America's American Mathematics Competitions. AMC logo.png